Sei sulla pagina 1di 16

http://www.rpmauryascienceblog.

com/

Q.1.

The induced emf in the rod causes a current to flow counter clockwise in the circuit. Because of
this current in the rod, it experiences a force to the left due to the magnetic field. In order to pull the
rod to the right with constant speed, the force must be balanced by the puller.
The induced emf in the rod is
|
= BLv = (0.25) (0.5)(4) = 0. 5 V
I = /R = 0. 5/3A from which,
F = iLBsin90o = (0.05)(0.5/3)(0.25) = (0.0625 /3)N

Q.2.

I = I0 (1 e-Rt/L)
1
1
U = LI2 , Umax = LI20
2
2
1
U = Umax
4
1 2 1 2
LI LI0
I = I0 / 2
2
8
I
From (i) 0 = I0 (1 e-Rt/L)
2
L

t = ln 2 = 5 ln2 = 3.47 s
R

Q.3.

B r2

Q.4.

Zero

Q.5.

= Bs = (8 + 5t) 102 800 104


d

= 5 102 800104 = 4 103 J/c
dt
(a) qMax = C = 10 106 4 103 = 4 108C
2

(i)

q
4 10 4 10

2c
2 10 10 6
(c) Plate B will be positively charged.

(b) Max energy stored is

Q.6.

A
B

= 8 1011 J

In accordance with law of potential distribution, for the given network,


dI
VA IR + E L
= VB
dt
And as here I is decreasing (dI/dt) is negative.
VB VA = 5 1 + 15 5 10-3 (103)

http://www.rpmauryascienceblog.com/
VB VA = 5 + 15 + 5 = 15 V
Q.7.

At t = 0 current through inductor will be zero.


Therefore current provided by the source I = E/R1 = 10/10 = 1 amp.
At t = , inductor will be shorted
E
Therefore current provided by the source I =
= 2 amp.
R1R2
R1 R2

T
4 I0
Iavrage
T
2
2
The torque applied by the magnetic force Fm about O is given as
dm = r(dFm) = r {i(dr) B}
I0

Q.8.
Q.9.

m =

r idrB;

E B 2

R
2R
B2 2

We obtain, m =
2R rdr

Putting i =

B2 2 r 2
m
2R 2

B2 4
4R

The external torque must be equal and opposite to the magnetic torque
B2 4
ext =
4R
B2 4 2
Power = ext =
4R
Q.10. Writing loop equation
4 1 10 10 103 103 VBA

E=4v

L=10mH

VAB 4 volts.

R=10
1amp
A

Q.11. The current at any time can be given by the expression, i = i0 (1 et/)
L 100mH
Where =

5 103 s
R
20
5 ln 2

1
1 i
10V

Where i0 =
0.5A
i 0.5A 1 e 5 = io 1 ln2 io 1 o
2
2
20
e

The energy stored =

1 2
1
1
1
J.
Li = 100 10 3 =
2
2
320
4

http://www.rpmauryascienceblog.com/
Q.12. Induced emf in the rod = BL (v cos )
BLv
i = /R =
cos
R
BLv

F = BiL = BL
cos
R

for uniform velocity force on rod up the plane = force on


rod down the plane
B2L2 v

cos2 mgsin
R
Rmg sin
v= 2 2

B L cos2
Rmg
v = 2 2 tan sec
BL
Q.13. (b) d = Rd
de = Rd . V sin . B
= RVB cos d
/2

e=

Mg cos

v
Mg sin

[ = 90 - ]

Mg

/ 2

/2

/2
cos = RVB [sin ] / 2

de = RVB

v cos

/ 2

= 2RVB.

Q.14. (a) =

B.d s

(b) e = -

= B.S

d B.dS

dt
dt
e

e = +BNlv

BN b
b
2b

3b

4b

x
b

2b

3b

4b

e = -BN v

Q.15. The induced emf in the rod is


|
= BLv = (0.25) (0.5)(4) = 0. 5 V
I = /R = 0.1 A
F = ILBsin90o = 1/80 N
Thus the total force acting on the rod on left is 1/40 N.
Hence 1/40 N of force is to be applied on the rod in the right side to move it with a constant velocity.
Q.16. The current at any time can be given by the expression, i = i0 (1 et/)
L 100mH
Where =

5 103 s
R
20
5 ln 2

i 0.5A 1 e 5

1
1 i

= io 1 ln 2 io 1 o
2 2
e

http://www.rpmauryascienceblog.com/
Where i0 =

10V
0.5A
20

The energy stored =


=

1 2
Li
2

1
1
100 10 3
2
4

1
J
320

Q.17. As the current in the spokes would be flowing in radially inward fashion , the force on each spoke
due to magnetic field directed into the plane of figure, turns out towards right, the initial torque
would be anticlockwise and hence the rotation.
Let i1 , i2 , i3 , i4 , i5 and i6 be the current through each spoke.
Consider the spoke through which current I1 flows.

For an element i dx at distance x from the centre, the magnetic force = i1dx B .
Hence the torque of this force = xi1dxB n
(Where n is unit vector in the direction of torque)
As torque for each spoke has the same direction ,
l
IBl 2
Total initial torque = Bni1 i2 ........ i6 xdx =
n 0.06Nmn .
2
0

Q.18. The loop equation


di
L
iR 0
dt
At t = 0, i = 0,
di
12 - 0.008
0
dt
di
12
= 2500 A/s

dt 0.008
di
Where
= 500A/s, an equation yields,
dt
12 - (0.008)(500) - 6i = 0
6 i = 12 - 4 i = (4/3) A
di
For the final current
= 0, and
dt
L(0) - 6 I
F = 0
IF = 2A
Q.19. The induced emf in the rod causes a current to flow counter clockwise in the circuit . Because of
this current in the rod, it experiences a force to the left due to the magnetic field. In order to pull the
rod to the right with constant speed, the force must be balanced by the puller.
The induced emf in the rod is
= BLv = (0.25)
| (0.5)(4) = 0. 5 V
I /R = 0. 5/3A from which, =
F = iLBsin90o = (0.05)(0.5/3)(0.25) = (0.0625 /3)N
Q.20. (a) Magnetic induction due to large loop at its centre is

http://www.rpmauryascienceblog.com/
B=

0ib
2b

i
B = B.A 0 b a2 cos
2b
where is angle between loops and = t
d a 2 0ib
induced emf = sin t

dt
2b
a20ib
current ia =

sin t
R 2bR
d a 2 0
(b) Mutual inductance M =

cos t
dt
2b
d
d
= (Mia )
dt
dt
from (i)
d a 2 0
a 2 0ib sin t
=
cos t.

dt 2b
2bR

and

(i)

ib a20 d sin 2t

R 2b dt 2

=-

i a20
cos 2t
2
= - b

R 2b
2
2

i a2 0
cos 2t
= - b
R 2b

Q.21. A conducting rod 'OA' of mass 'm' and length 'l' is kept rotating in a vertical plane . . . . . any other
resistance.
(a)

1 2
Bl = e
2

(b) E = iR + L

di
dt

Rt
log(E iR) c
L
E iR = EeRT/L
E
i=
1 e RT / L
R
1 1

i = Bl2 1 eRT / L
R 2

dt
di

L
E iR

Bl2
at t steady state
2R
Power = Torque ()
i2 R = J

i=

http://www.rpmauryascienceblog.com/
i2R B2l42R

+ torque due to weight of the rod

4R2
B 2l4
B 2l4
J=
+ torque due to weight of the rod =
+ Mg (/2)cos t
4R
4R
J=

Q.22. Let the velocities of the two rods at time t be v1 i and v2 i . Net induced emf in the circuit = B(v2
v 1)
B(v 2 v 1 )
Induced current =
(clockwise)
R

B2 ( v 2 v1 )
As a result, a force F =
i acts on the rod of mass m and a force
R

B2 ( v 2 v1 )
F =i acts on the rod of mass 2m.
R
Equations of motion of m, 2m, and 3m are
dv
B2 2 ( v 2 v 1 )
m 1 =
. . .. (i)
dt
R
dv
B2 2 ( v 2 v 1 )
2m 2 = T . . .. (ii)
dt
R
dv
3m 2 = 3mg T
. . .. (iii)
dt
(T being the tension in string)
Adding (ii) and (iii),
dv
B22 ( v 2 v1 )
5m 2 = 3,mg . . .. (iv)
dt
R
From equation (i) and (iv)
d
6B2 2 ( v 2 v 1 )
5m ( v 2 v1 ) = 3mg dt
R
Solving this equation, we get
6B 2 2 t

mgR
5mR
v2 v1 =
1

2B2 2

Putting this in (i)


6B 2 2 t

dv 1
mg
m
=
1 e 5 mR

dt
2

Integrating we get
6 B 2 2 t

g
5mR
v1 = t
1 e 5 mR

2 6B2 2

Putting values
v1 = 5 [t 5 (1 e-(t/5) ) ] m/s
where t is in seconds.

http://www.rpmauryascienceblog.com/
a x

Q.23. =

B0
a x
ad B0a n

= B0a n1
vt

vt
a
= B B0 a
dt
a vt vt 2
B0 a

I=
.
4(a vt )t
1.25

Q.24. Q =

0 (3 4t 2 )
.
0.25 dx
2
x

= 0 (3 4t 2 ) 0.25 ln (1.25)
2
dQ 0
e=
0.25h(1.25) 8t
dt
2
di 0 t ln(1.25)

dt
L
0 ln(1.25 ) 2
i=
t
10

Q.25. (a)The rotation of the ring about point P


generates an emf. The ring within P & Q is
equivalent to a rod of length PQ.
Now PQ =

a 2 a2 a 2

The emf between P and Q is given

by
2
1
B a 2 = Ba2
2
(b) As the resistance between P & Q is
R. Then the current
Ba 2
I=

R
R

Q.26. From faraday's law of emf


(a) e = -

d
(BA )
dt

with A = R2
B = B0sin 2 ft
so e = - 22R2 f B0 cos 2 ft.
(b) Use R =
R =

2R
a

L
, (Here R is resistance
a

http://www.rpmauryascienceblog.com/
(c) I =

e
and P = I2R.
R/

Q. 27. (a) (i) F = BId


(ii) e = I(R+2x)
(b) If v is the velocity of the rod, the induced emf = Bvd
The current is given by,
e
Bvd
I=
=
R
(R 2x )
If I is constant,
R 2 x
v=
I
. . . (1)
Bd
F - Fm = ma or,
F = Fm + ma
2I dx
or,
F = Bid + m
Bd dt
2mI2
or,
F = Bid + 2 2 (R + 2x)
. . . (2)
B d

R 2 x
dw
2mI2
(c)
P Fv BId 2 2 (R 2 x )
I
dt
B d

Bd
= Power dissipated in circuit + Rate of increase of kinetic energy of rod
2mI3
= I2 (R + 2x) + 3 3 (R + 2x)2
B d
The fraction is,
I2 (R 2x ) 2mI(R 2x )
f=
1

P
B3d3

f = 0.961.
a x

Q.28. =

0I
Ia a x
ady 0 ln

2y
2 x

Ia
a
= 0 ln 1
2
vt

Ia vt
a
= B 0
dt
2 a vt vt 2
0Ia

I=
8 t a vt

v
y

dy

Q.29. At P due to current in (1), magnetic field is in upward direction and due to current in (2), magnetic
field is downward direction.
At Q due to current in (1) magnetic field is downward and due to current in (2), magnetic field is in
upward direction.
20 o 30
Therefore at P, B1 - B2 = o
= 2 10-5 N/Ampmeter, along positive z-axis

2 0.1 2 0.3
o 20 o 30
at Q, B1 + B2 =

= 1 10-4 N/Ampmeter, along negative z-axis


2 0.1 2 0.3

http://www.rpmauryascienceblog.com/
at R,

B2 - B1 =

o 30 o 20

= 4.7 10-5 N/Ampmeter, along positive z-axis


2 0.1 2 0.3

Q.30. (a) Applying KVL to the circuit at time 't'


0.2 di
2i +
= 20
10 dt
solving this differential equation :
i = 10 (1- 0.1e-100t )
d
(b) EMF induced = dt
1 d
current induced =
=i
R dt
d

idt =
R

P
5A

1m

3m

?
d
Integrating, idt
= net change in flux /R
R
net change in flux
Hence charge flown =
R
(1)(5) 3
4
Now net change in flux = i - f = 0
ln ln Weber
2
3
2
charge flown = 1.17 10-7 coulomb.
Q.31. The loop equation
di
L
iR 0
dt
At t = 0, i = 0,
di
60 - 0.008
0
dt
di
60

= 7500 A/s
dt 0.008
di
Where
= 500A/s, an equation yields,
dt
60 - (0.008)(500) - 30i = 0
30 i = 60 - 4 i = 1.867 A
di
For the final current
= 0, and
dt
L(0) - 30 I
F = 0
IF = 2A
Q.32. (a) Given L = 2.0 mH, C = 5.0 F and Qmax = 200C
Let q be the charge on the capacitor at any time t and I be the current flowing in te
circuit.
Then

http://www.rpmauryascienceblog.com/
q
dI
L
0
C
dt
dI
q
100 10 6

6
dt LC
(2 10 3 )(5 10 6 )
= 104 amp/sec.
(a)

(b) When Q = 200 C, then the total energy of the circuit is


stored in the capacitor i.e., no current flows through the
circuit. Hence current I in the circuit is zero.
I=0
(c) Let I0 be the maximum value of the current. Then
2
1 2 1 Qmax
LI0
2
2 C
Q2
Q2
or I20 max or
I0 max
LC
LC
I0 = 2 amp
By conservation of energy
2
1 Qmax
1 Li20 1 q2

2 C
2 4 2 C
Q
3

Q = max
= 100 3 C
2
x

Q.33. X = a + vt, =

0Id

2x dx
a

0 I
2

x
dln
a

0Idv 0
d
emf =

dt 2(a vt)
0 Idv 0
Current I =
2R(a vt)

F = ILB =

Q.34.

20I2 d2 v 0
42 (a vt)2 R

1 2 1 2
Li0 Cv 0
2
2
C
i0 = v 0
L
q
5.0 10 6
v0 = 0
1.25 10 2 volt
4
C 4.0 10
4.0 104
-2
8.33 10 4 A
i0 = 1.25 10
0.09
umax =

1 2 1 2
Li0 Cv 0 3.125 10 8 J
2
2
2

3.125 10-8 =

8.33 104
1
1
q2
(0.09)

2
2
2 (4.0 104 )

q = 4.33 10-6 C.

dI
dt

+
q

http://www.rpmauryascienceblog.com/
Q.35. Vel. Comp along x-axis v cos
will remain
constant
m
x v cos .
2qB
due to vel comp along y axis v sin it will move
mv sin
in a circular path of radius R
qB
y-Co-ordinate
y R sin R sin t

Y-axis

- Z-axis

qB m
mv sin
sin
.

qB
m 2qB

mv sin
mv sin
.sin / 2
qB
qB
Z - coordinate
z R R cos

R 1 cos

mv sin

1 cos
qB
2
mv sin

qB

Q.36. VB sin 180


VB sin
VB sin
I
R
R
2B 2 V sin
Fmag IB
R
2 2
2
B V sin
Now
mg sin 0
R
mgR
or B 2
V sin

Q.37. (a) In one complete cycle


Iav = 0
/2

Irms =

2I0

t dt

/2

dt
0

Irms =

I0
3

0.5

(b) U =

v2
dt
R

0.5

1 1
(10 sin 2t)2
dt =
= 0.025 J.
3
10 4
10

F
F sin
90-

B
mg sin

http://www.rpmauryascienceblog.com/
Q.38. (i) Impedance Z =

R 2 (XL X C )2 64 36 10

v 220

22A
z
10
| XL XC | 6 3
(iii) tan =

R
8 4
= 370 (current leads)
(iv) Power factor, cos = R/Z = 0.8

(ii) Current =

Q.39. Emf induced in the coil, E = -

d
dB
= -nA
dt
dt

where n = number of turns, A = area, B = magnetic field.


Current in the coil i1 =

E
nA dB
0.5(0.8 0)

- 1.25 A
R
R dt
1.6(0.2)

Current in the coil when magnetic field is decreased uniformly from 0.8 T to 0 in 0.2 sec.
i2 = -

nA dB
0.5 (0 0.8)
= 1.25 A

R dt
1.6 0.2

Power dissipated P = i2R = (1.25)2 (1.6) = 2.5 W


Graph showing variation of current
Current (i)

Graph showing variation of power


Power

2.5 W

1.25 A
0.2 s.

0.4 s.

0.8 s.

0.8 s.

-1.25 A

Heat dissipated in the time interval from 0 to 0.4 sec. = i12 Rt + i 22 Rt


= 2 (1.25)2 (1.6) (0.2) = 1 J
Total heat dissipated when the cycle is repeated 12000 times
H = (12000) (1) J = 12000 J
Let T = increase in temperature
Therefore H = (m1c 1 + m2c 2)T
T = 5.63 0C
Therefore final temperature T = 35. 63 0C

Q.40. Area (PQRS) =

2
3

(a x )dx

Flux over PQRS = d =

2
3

(a x )dx
3
a
2

Flux over whole ,

= d =

0 2I
4 d x

a x

3 d x
0I

SR
A B
d

x Q
P

dx

http://www.rpmauryascienceblog.com/

0I
=

I=

3
2

dx

ad
dx
d x

e
0I
3a
3
d

)
a cos t

(a d) log(1
dt
R
2d
2
3R
R

Q.41. At t = t
b
V a V b = Vc V d
di
dx
L
B
dt
dt
Ldi = B dx
x=0
li = B x
x
B
i= x
L
c
net force on the connector
Fnet = mg Fm
dv
m
mg iB
dt
B2 2
= mg x
L
B22
dv
x
g

dt
mL
d2 v
B 2 2 dx

.
mL dt
dt 2
d2 v

2 v
2
dt
B
=
mL
dv
v = v 0 sin t,
v 0 cos t
dt
dv
at t = 0,
=g
dt
g = v0
g
v0 =

g mL
=
B
g mL
B
so v = v 0 sin t [where v0 =
and =
]
B
mL
dx

v 0 sin t
dt

L
i

t=0
v= 0
Fm

d
mg

t=t

http://www.rpmauryascienceblog.com/
x

dx u 0 sin t dt
0

x=

v0
[ - cos t] 0t

v0
(1 cos t)

v
x = 0 1 cos t

x=

di
Bv
dt
di
dx
L B
dt
dt
B
i=
x
L
Magnetic force on the rod,
B2 2
Fm = iB =
x
L
d2 x
B2 2
m 2
x
L
dt
d2 x
B22
x

dt 2
m2L
B
=
.
mL

Q.42. L

(Force is in opposite direction of v )

Q.43. = 1m, mbar = 1kg , B = 0.6 T


Let v = terminal velocity of the bar L.
The induced emf developed across the bar is
= Bv
P
P
Current in R1 and R2 are I1 = 1 ;
I2 = 2

Total current through bar is I = I1 + i2 = (P1 + P2)/


Force experienced by bar is F = IB
When the bar attains a constant velocity we have
mg
mg = IB

I=
B
mg P1 P2
from (i) and (ii)

B
Bv
P1 P2 0.76 1.2
v=
= 1.0 m/s

mg
0.2 9.8
= Bv = 0.6 1 1 = 0.6 v
2 (0.6)2
R1 =

0.474
I1 P1
0.76
R2 =

2 (0.6)2

= 0.3
P2
1.2

(i)

(ii)

v
m
C

http://www.rpmauryascienceblog.com/
Q.44. (a) Equivalent Inductance =
f=

1
2( 3 / 2)LC

L.L
3
+ L= L
LL
2

(b) (I) Applying kirchooff's second law to meshes


(1) and (2)
2I2 + 2I1 = 12 - 3 = 9
and 2I1 + 2(I1 -I2) = 12
solving I1 = 3.5 A, I2 = 1A
P.D.between AB = 2I2 + 3 = 5 volt
dQ 2
Rate of production of heat
i1R1 24.5 (J)
dt
E
1 e Rt / L = i0 1 eRt / L
R
Rt
i = i0/2
= loge2
L
t = 0.0014 sec.
1 2
Energy stored =
Li
2
= 0.00045 (J)

(ii) i =

Q.45. (a) de = B (x) dx


Br 2
e=
2
(b) i = i0 (1 - e RT / L )
Br 2
i=
1 e Rt / L
2R

R3
2 (1)
I1R1 I1
I2

R1

E1

12V
E2

S
A

E2 = 3V

(I1-I2)
(2)
R2 B

3V

R2 = 2

R4 = 3

10 mH

A
dx
R

L
O

i0 = e/R
i

(c) In steady state, i = i0


Bet torque = mg (r/2) cos + Fx
=

I2

mgr cos
mgr cos
iB.xdx =

2
2
0
mgr cos t
1 2 4

B r
2
4R

dx
r

B r
.Bxdx
2R
0

F
mg

http://www.rpmauryascienceblog.com/

Q.46. v Yi ;

d dyj

I
d
B


0Ik
; d v Ydyk
L
2[d Y]
2

0IYdy
B.(d v)
L
2[d Y]
2
L/2
I
I
Ydy
L
L L
vB v0 = 0
= 0 d ln 1
2
2 2d 2
0 [d L y]
2

y
dy
L

Y
O

x
z

Q.47. The repulsive force on the side ps of the current-carrying loop, due i1
p
to current i1 is
ii L
20 16 0.15
F1 o 1 2 (2 10 7 )
2.4 10 4 N
2 d
0.04
i2
This force will be towards RHS and to the current-carrying wire
ps.
s
Similarly, the attractive force acting on the side qr of
the loop, due to current I1 is (Here R = d+b = 10 cm = 0.1 meter)
d
20 16 0.15
7
F2 (2 10 )
0.10
4
0.96 10 N .
Direction of this force will be towards LHS and to current-carrying wire qr.
The forces acting on the sides pq and rs of the loop will be equal and opposite.
Thus net force on the loop = F1 F2 = (2.4 0.96) 104 = 1.44 104 N
(Acting away from the current-carrying wire)

r
b

When the direction of current in the loop becomes clockwise, the net force on the loop remain
same, but its direction now becomes towards the current-carrying wire.

Potrebbero piacerti anche